K
Khách

Hãy nhập câu hỏi của bạn vào đây, nếu là tài khoản VIP, bạn sẽ được ưu tiên trả lời.

AH
Akai Haruma
Giáo viên
28 tháng 5 2018

Lời giải:

Để pt có hai nghiệm $x_1,x_2$ thì:

\(\Delta'=1-m>0\Leftrightarrow m< 1\)

Áp dụng định lý Viete cho pt bậc 2 ta có: \(\left\{\begin{matrix} x_1+x_2=2\\ x_1x_2=m\end{matrix}\right.\)

Khi đó:

\(N=x_1^4+x_2^4-2(x_1^3+x_2^3)+8m\)

\(=(x_1^2+x_2^2)^2-2(x_1x_2)^2-2[(x_1+x_2)^3-3x_1x_2(x_1+x_2)]+8m\)

\(=[(x_1+x_2)^2-2x_1x_2]^2-2(x_1x_2)^2-2[(x_1+x_2)^3-3x_1x_2(x_1+x_2)]+8m\)

\(=(4-2m)^2-2m^2-2(8-6m)+8m\)

\(=2m^2+4m=2(m^2+2m+1)-2=(m+1)^2-2\geq -2\) với mọi $m< 1$

Do đó \(N_{\min}=-2\) khi \(m=-1\)

5 tháng 6 2018

ta có : \(2\left(m^2+2m+1\right)-2=2\left(m+1\right)^2-2\) chứ không phải bằng \(\left(m+1\right)^2-2\)

Thứ sáu, em thi rồi mong mọi người giúp em giải đề thi này: Bài 1: Cho: \(P=\left(\dfrac{\sqrt{x}}{x^2-1}+\dfrac{\sqrt{x}}{\sqrt{x}-1}+\dfrac{\sqrt{x}}{\sqrt{x}+1}\right)\dfrac{x^2-1}{\sqrt{x}}-2\sqrt{x}^3-\sqrt{x}\) ( ĐK \(x0,x#1\) ) \(Q=x-1\) a) Rút gọn P. Tính P khi x = 9. b) Tính x khi \(2P-Q=0.\) c) Tìm GTNT của \(\dfrac{P}{Q}\) . Bài 2: Cho (P) \(y=\dfrac{x^2}{2}\) Và (d) \(y=3x+6\) a) Vẽ (P) và (d) trên cùng một phẳng. b) Cho (d1) \(y=kx+k^2+2\) ....
Đọc tiếp

Thứ sáu, em thi rồi mong mọi người giúp em giải đề thi này:

Bài 1: Cho:

\(P=\left(\dfrac{\sqrt{x}}{x^2-1}+\dfrac{\sqrt{x}}{\sqrt{x}-1}+\dfrac{\sqrt{x}}{\sqrt{x}+1}\right)\dfrac{x^2-1}{\sqrt{x}}-2\sqrt{x}^3-\sqrt{x}\)

( ĐK \(x>0,x#1\) )

\(Q=x-1\)

a) Rút gọn P. Tính P khi x = 9.

b) Tính x khi \(2P-Q=0.\)

c) Tìm GTNT của \(\dfrac{P}{Q}\) .

Bài 2: Cho (P) \(y=\dfrac{x^2}{2}\) Và (d) \(y=3x+6\)

a) Vẽ (P) và (d) trên cùng một phẳng.

b) Cho (d1) \(y=kx+k^2+2\) . Tìm k để (d), (d1) và (d2) \(y=x+2\) đồng quy.

c) Tìm điểm cố định của (d1).

Bài 3: Cho phương trình:

\(x^2-2\left(m+1\right)x+m-1=0\)

a) Chứng minh phương trình luôn có hai nghiệm phân biệt.

b) Gọi x1,x2 là hai nghiệm của phương trình. Tìm m để x1,x2 lần lượt là chiều dài và chiều rộng của hình chữ nhật sao cho bình phương nữa chu vi cộng cho diện tích bằng 5.

c) Tìm m để :

\(\left(3x_1-x_2\right)^2+\left(3x_1-x_2\right)-6=0\) ( ĐK \(\sqrt{x_1}>\sqrt{x_2}\) )

Bài 4: Tìm hai số tự nhiên x,y biết: bình phương của hai số đó cộng lại bằng 5. Và 1009x - 2018y =0 thỏa mản (x2-3y)2018=1.

Bài 5: Cho đường tròn tâm O, bán kính R. Từ điểm I ngoài đường tròn kẻ hai tiếp tuyến IA,IB với đường tròn. OI cắt AB tại H. Kẻ điểm E đối xứng với H qua O. Từ E kẻ đường thẳng cắt IA và IB lần lượt tại hai điểm M,N.

a) Tính AH theo R khi AI=3R. Và diện tích tam giác ABI.

b) Chứng minh rằng tứ giác IAOB nội tiếp. Và Tứ giác ABNM là hình thang cân.

c) Chứng minh: BI.ME=IM.HB

d) Chứng minh A,O,N nằm trên một đường thẳng. Và Tam giác AEB là tam giác cân.

5
AH
Akai Haruma
Giáo viên
29 tháng 5 2018

Bài 1:

a) Có: \(P=\left(\frac{\sqrt{x}}{x^2-1}+\frac{\sqrt{x}}{\sqrt{x}-1}+\frac{\sqrt{x}}{\sqrt{x}+1}\right).\frac{x^2-1}{\sqrt{x}}-2\sqrt{x^3}-\sqrt{x}\)

\(=\left(\frac{\sqrt{x}}{x^2-1}+\frac{x+\sqrt{x}+x-\sqrt{x}}{(\sqrt{x}-1)(\sqrt{x}+1)}\right).\frac{x^2-1}{\sqrt{x}}-2\sqrt{x^3}-\sqrt{x}\)

\(=\left(\frac{\sqrt{x}}{x^2-1}+\frac{2x}{x-1}\right).\frac{x^2-1}{\sqrt{x}}-2\sqrt{x^3}-\sqrt{x}\)

\(=1+\frac{2x(x+1)}{\sqrt{x}}-2\sqrt{x^3}-\sqrt{x}\)

\(=1+\sqrt{x}\)

Khi \(x=9\Rightarrow P=1+\sqrt{9}=4\)

b) \(2P-Q=0\Leftrightarrow 2(1+\sqrt{x})-(x-1)=0\)

\(\Leftrightarrow -x+2\sqrt{x}+3=0\)

\(\Leftrightarrow (3-\sqrt{x})(\sqrt{x}+1)=0\)

\(\Leftrightarrow 3-\sqrt{x}=0\Leftrightarrow x=9\)

c) Phần c là tìm GTNN của \(\frac{P}{Q}\) hả em?

\(\frac{P}{Q}=\frac{\sqrt{x}+1}{x-1}=\frac{1}{\sqrt{x}-1}\)

Với \(x>0\) càng tiến gần tới 1 theo chiều từ trái sang phải thì \(\sqrt{x}-1\) càng nhận giá trị âm càng lớn. Do đó \(\frac{1}{\sqrt{x}-1}\) càng nhận giá trị âm càng nhỏ

Do đó P/Q không có GTNN

AH
Akai Haruma
Giáo viên
29 tháng 5 2018

Bài 2:

Hình vẽ:

Violympic toán 9

28 tháng 5 2018

∆ABC: BC=√2, ^A=90°

O(tam (O) noi tiep

S=S∆abc=S∆boc+S∆boa+S∆aoc

s1=S∆boc=1/2.r.BC=√2/2

S2=S∆boa+S∆aoc=1/2.r.(AB+AC)

4S2^2=(AB+AC)^2=AB^2+AC^2+2AB.AC=BC^2+2S

=2+2S,

S=2S2^2 -1

S=s1+s2=√2/2+s2

2s2^2 -s2-1-√2/2

1+8+4√2=9+4√2=(2√2+1)^2

s2=(1+2√2+1)/4=(1+√2)/2

S=(√2+1+√2)/2=√2+0,5

AH
Akai Haruma
Giáo viên
28 tháng 5 2018

Lời giải:

Gọi độ dài hai cạnh tam giác vuông là $a,b$ thì cạnh huyền là:

\(\sqrt{a^2+b^2}=\sqrt{2}\) \(\Rightarrow a^2+b^2=2\)

Ta có công thức sau:

\(S_{ABC}=pr\) (p là nửa chu vi)

\(\Leftrightarrow \frac{ab}{2}=\frac{a+b+\sqrt{a^2+b^2}}{2}.1\)

\(\Leftrightarrow ab=a+b+\sqrt{2}\)

\(\Rightarrow ab-\sqrt{2}=a+b\)

\(\Rightarrow (ab-\sqrt{2})^2=(a+b)^2=a^2+b^2+2ab\)

\(\Leftrightarrow a^2b^2+2-2\sqrt{2}ab=2+2ab\)

\(\Leftrightarrow a^2b^2-(2\sqrt{2}+2)ab=0\)

Vì $ab\neq 0$ nên \(ab=2\sqrt{2}+2\Rightarrow S_{ABC}=\frac{ab}{2}=\sqrt{2}+1\)

27 tháng 5 2018
https://i.imgur.com/lTmPmaR.jpg
28 tháng 5 2018

B=1; B-1=0

B-1=[√x/(√x-1)-2]+[2√x-1)/(√x+1)-1]+[6/(1-x)+2]

=(-√x+2)/(√x-1)+(√x-2)+(-2(x-4))/(1-x)

=(-√x+2)[1/(√x-1)-2/(√x+1)+2(√x+2)/(1-x)]

x>0=>[....].≠0

-√x+2=0

x=4.

AH
Akai Haruma
Giáo viên
26 tháng 5 2018

Lời giải:

Ta có:

\(x^2+1-2x=(x-1)^2\geq 0\Rightarrow 2x\leq x^2+1(1)\)

\(x^2+y^2=1\Rightarrow y^2=1-x^2\leq 1\Rightarrow -1\leq y\leq 1\)

Do đó \(y^3-y^2=y^2(y-1)\leq 0\Rightarrow y^3\leq y^2(2)\)

Từ \((1);(2)\Rightarrow P=2x+y^3\leq x^2+1+y^2=2\)

Vậy \(P_{\max}=2\). Dấu bằng xảy ra khi \((x,y)=(1,0)\)

AH
Akai Haruma
Giáo viên
23 tháng 5 2018

Lời giải:

Gọi $d$ là ước chung lớn nhất của $a,b$

Khi đó, đặt \(\left\{\begin{matrix} a=dx\\ b=dy\end{matrix}\right.(x,y)=1\)

Ta có: \(ab(a+b)\vdots a^2+ab+b^2\)

\(\Leftrightarrow dxdy(dx+dy)\vdots (dx)^2+dxdy+(dy)^2\)

\(\Leftrightarrow dxy(x+y)\vdots x^2+xy+y^2\)

Do $x,y$ nguyên tố cùng nhau nên :

\((x,x^2+xy+y^2)= (y,x^2+xy+y^2)=(x+y,x^2+xy+y^2)=1\)

Suy ra \(d\vdots x^2+xy+y^2\)

\(\Rightarrow d\geq x^2+xy+y^2\)

\(\Rightarrow d^3\geq a^2+ab+b^2\)

Mà với $a,b$ nguyên dương phân biệt thì \(a^2+ab+b^2\geq 3ab>ab\)

Do đó \(d^3>ab(1)\)

Mặt khác: $a,b$ nguyên dương phân biệt kéo theo $x,y$ nguyên dương phân biệt nên \(|x-y|\geq 1\)

\(\Rightarrow |a-b|=d|x-y|\geq d(2)\)

Từ \((1);(2)\Rightarrow |a-b|^3>ab\Rightarrow |a-b|>\sqrt[3]{ab}\)

Ta có đpcm.

AH
Akai Haruma
Giáo viên
23 tháng 5 2018

Lời giải:

\(a,b,c\in [-2;5]\) nên:

\(\left\{\begin{matrix} (a+2)(a-5)\leq 0\\ (b+2)(b-5)\leq 0\\ (c+2)(c-5)\leq 0\end{matrix}\right.\) \(\Leftrightarrow \left\{\begin{matrix} a^2\leq 3a+10\\ b^2\leq 3b+10\\ c^2\leq 3c+10\end{matrix}\right.\)

\(\Leftrightarrow \left\{\begin{matrix} a^2\leq 3a+10\\ 2b^2\leq 6b+20\\ 3c^2\leq 9c+30\end{matrix}\right. \)

Do đó:

\(a^2+2b^2+3c^2\leq 3(a+2b+3c)+60\)

\(a+2b+3c\leq 2\)

\(\Rightarrow a^2+2b^2+3c^2\leq 3.2+60=66\)

Ta có đpcm

Dấu bằng xảy ra khi \((a,b,c)=(-2,5,-2)\)

20 tháng 5 2018

x,y€0;1]

(x-1)(y-1)≥0

xy-(x+y)+1≥0

3xy-3(x+y)+3≥0:; -2(x+y)+3≥0

(x+y)≤3/2

x+y=3xy=>9(xy)^2-4(xy)≥0=> xy≥4/9

=>(x+y)€[4/3;3/2]

P=x^2+y^2-4xy=(x+y)^2-6xy=(x+y)^2-2(x+y)=[(x+y-1]^2-1

Pmin=(4/3-1)^2-1=1/9-1=-8/9

khi x+y=4 /3; xy=4/9

x=y=2/3

Pmax=(3/2-1)^2-1=1/4-1=-3/4

khi x or y =1

(x,y)=(1,1/2);(1/2;1)

20 tháng 5 2018

\(P=x^2+y^2-4xy\)

\(P=\left(x+y\right)^2-2xy-4xy\)

\(P=\left(3xy\right)^2-6xy\)

\(P=\left(3xy\right)^2-2.3xy.1+1-1\)

\(P=\left(3xy-1\right)^2-1\ge-1\)

dấu \("="\) xảy ra \(\Leftrightarrow3xy-1=0\Leftrightarrow xy=\dfrac{1}{3}\)

vậy MIN \(P=-1\Leftrightarrow xy=\dfrac{1}{3}\)

19 tháng 5 2018

qua vo van

19 tháng 5 2018

Thôi làm luôn nãy h chém nhiều mỏi tay quá. Bổ sung điều kiện a;b;c>1

\(\dfrac{4a^2}{a-1}+\dfrac{5b^2}{b-1}+\dfrac{3c^2}{c-1}\ge48\)

\(\Rightarrow\left(\dfrac{4a^2}{a-1}-16\right)+\left(\dfrac{5b^2}{b-1}-20\right)+\left(\dfrac{3c^2}{c-1}-12\right)\ge0\)

\(\Rightarrow\dfrac{4a^2-16a+16}{a-1}+\dfrac{5b^2-20b+20}{b-1}+\dfrac{3c^2-12c+12}{c-1}\ge0\)

\(\Rightarrow\dfrac{4\left(a-2\right)^2}{a-1}+\dfrac{5\left(b-2\right)^2}{b-1}+\dfrac{3\left(c-2\right)^2}{c-1}\ge0\) (đúng)

Dấu "=" khi \(a=b=c=2\)

AH
Akai Haruma
Giáo viên
16 tháng 5 2018

Lời giải:

Đặc điểm đặc biệt của một số lập phương là chia $7$ dư $0,1$ hoặc $6$ nên ta sẽ sử dụng mod 7 trong bài này.

Ta thấy: \(2^n=x^3+3367\equiv x^3\pmod 7\)

\(x^3\equiv 0,1,6\pmod 7\Rightarrow 2^n\equiv 0,1,6\pmod 7\)

Ta thấy \(2^3\equiv 1\pmod 7\) nên xét các TH sau:

+) Nếu \(n=3k\Rightarrow 2^n=2^{3k}\equiv 1\pmod 7\)

+) Nếu \(n=3k+1\Rightarrow 2^n=2^{3k+1}=2^{3k}.2\equiv 2\pmod 7\)

+) Nếu \(n=3k+2\Rightarrow 2^n=2^{3k+2}=2^{3k}.4\equiv 4\pmod 7\)

Từ các TH trên suy ra \(n=3k\) là th duy nhất có thể xảy ra.

Khi đó:

\(x^3+3367=2^{3k}=(2^k)^3\)

\(\Leftrightarrow 3367=(2^k)^3-x^3\)

\(\Leftrightarrow 3367=(2^k-x)(2^{2k}+x.2^k+x^2)\)

Đây là dạng pt tích đơn giản . Thử các TH ta suy ra \(x=9, k=4\rightarrow n=12\)

Vậy \((x,n)=(9,12)\)

23 tháng 5 2018

PT đã cho suy ra x3≡2n(mod7) (1)

Nếu n không chia hết cho 3 thì 2n khi chia cho 7 chỉ có thể cho số dư là 2,4,7 trong khi đó x3 khi chia cho 7 chỉ có thể cho số dư là 0,1,6 . Trái với (1)

Vậy n=3m với m là số nguyên dương . Thay vào PT đã cho

x3+3367=23m⇔(2m−x)[(2m−x)2+3x.2m]=3367 (2)

Từ (2) suy ra 2m−x là ước của 3367

Mà (2m−x)3<23m−x3=3367

Nên 2m−x∈1;7;13

Đến đây xét 3 TH là OK

Nghiệm là (x;n)=(9;12)